2
$\begingroup$

I am going through Spivak's Calculus right now and I am trying the exercises in Chapter 5. Spivak gives the following definition of the limit $L$ of a function as $x$ approaches $a$:

$$\forall \epsilon>0, \exists\delta>0:0<\left\lvert x-a \right\rvert<\delta \implies \left\lvert f(x)-L \right\rvert<\epsilon.$$

To prove that a limit doesn't exist, acoording to Spivak, one must use the following negation:

$$\forall L, \exists\epsilon>0:\forall \delta>0, \exists x: 0<\left\lvert x-a \right\rvert<\delta \ \text{and} \left\lvert f(x)-L \right\rvert\geqslant\epsilon.$$

Now my question is, since $\delta$ can depend on $\epsilon$ when proving that a limit exists, why haven't I seen anyone pick $\epsilon$ based on $\delta$ when doing the opposite? What I have noticed in those cases is that the author would pick an abitrary number $\beta$ in place of $\delta$, find a suitable $\epsilon$ in terms of that number and then pick $0<\left\lvert x-a \right\rvert < \min(\beta, \delta)$ to obtain the desired contradiction for any $\delta>0$

$\endgroup$
12
  • 2
    $\begingroup$ Wrong negation. $\endgroup$ Commented Jun 20 at 10:22
  • $\begingroup$ $\neg(p\to q)=\neg(\neg p\lor q)=p\land\neg q$ $\endgroup$ Commented Jun 20 at 10:26
  • $\begingroup$ Thank you! I corrected the mistaken negation. $\endgroup$ Commented Jun 20 at 10:27
  • 1
    $\begingroup$ Of course you choose $\delta$ according to the choice of $\epsilon$ in showing that a limit exists. How else would you do it? $\endgroup$ Commented Jun 20 at 10:38
  • 1
    $\begingroup$ When you have a $\forall$ followed by an $\exists$, the $\exists$ variable depends on the $\forall$ variable. So in the first case $\delta$ depends on $\epsilon$, whereas in the second $\epsilon$ depends on $L$ while $x$ depends on $\delta$ (and also on $L$ and $\epsilon$, since those are used to "set up" the statement about all $\delta$). $\endgroup$ Commented Jun 20 at 13:21

2 Answers 2

3
$\begingroup$

When you have a statement with quantifiers, like $(\forall \epsilon>0) (\exists \delta>0) \ldots,$ the existentially quantified variables (e.g. $\exists \delta$) can depend on universally quantified variables (e.g. $\forall \epsilon$) that precedes them. So $\delta$ can depend on $\epsilon$ here, but not reversely.

For $(\forall L)(\exists\epsilon>0)(\forall\delta>0)\ldots$ the above rule gives that $\epsilon$ can depend on $L$ but not on $\delta.$

$\endgroup$
3
$\begingroup$

When you use, $$\forall L\in\mathbb R,\exists\varepsilon>0\text{ s.t. }\forall\delta>0,\exists x\in\mathbb R\text{ s.t. }0<|x-a|<\delta \text{ and }|f(x)-L|\ge \varepsilon$$ what's happening is, given your arbitrary $L\in\mathbb R$, the statement guarantees the existence of a $\varepsilon>0$ such that the rest of the statement holds. This $\varepsilon$ should only depend on $L$, since we only told it our arbitrary $L$.

$\endgroup$
0

You must log in to answer this question.

Start asking to get answers

Find the answer to your question by asking.

Ask question

Explore related questions

See similar questions with these tags.